Consequent upon the ban imposed by the Government on smoking

This topic has expert replies
Moderator
Posts: 772
Joined: Wed Aug 30, 2017 6:29 pm
Followed by:6 members
Consequent upon the ban imposed by the Government on smoking in public places there has been a decline in the sales of tobacco products, with every seller reporting a reduction of twenty percent or more in sales Volume. Smoking appears to be losing its appeal. The government claims that this is solely due to its ban.

Which of the following, if true, would weaken the Government's claim?

A. Though the number of smokes may have come down, there is an increase in the number of Smokers.
B. People continued to smoke in the areas other than those notified as 'No smoking zones'
C. There has been ambiguity regarding the classification of areas as 'public areas' and 'other places'.
D. Prices of cigarettes have gone up by over fifty percent after the ban was imposed.
E. After the Government ban, the local police have been following the smoking ban strictly.
OA is d

Legendary Member
Posts: 2214
Joined: Fri Mar 02, 2018 2:22 pm
Followed by:5 members

by deloitte247 » Sun Apr 22, 2018 9:22 am
Option A; this somewhat supports the 'Government Claim' that is, it does not totally 'weakens' the claim of the government. The examiner wants a claim that 'weakens' in order words a statement that'goes against what was contained in the original statement' This option is wrong.

Option B; If this statement is true it does not in anyways weakens the writers statement. The use of 'smoking zones' was not used thus this non sequitor. This option is incorrect.

Option C; Yes, there may be ambiguity, true or perhaps maybe false but does this truly weakens the writers statement? No. Let's go for other options. This option is incorrect!!!!

Option D; Let's take a critical look at this, 'prices of cigarettes have gone up over fifth percent' this alone weakens, go against, and congrats the writers statement. Thus, if this statement is true it weakens the writers statement. Hence this option is correct.

Option E; This does not in anyways connects into the writers statement in the above question. Local Police were not mentioned. It wouldn't weaken it a bit!!! This option is incorrect!!!